Copernico Felinis
Copernico Felinis
  • 24
  • 14 009
Voltage for grownups
A description of what voltage really is when we allow for certain circuit parameters to vary with time. This video is about voltage in stationary circuits subject to quasi-static conditions.
00:00 Intro
02:50 Voltage is work per unit charge
07:38 The electromagnetic force
09:18 Motional and non-motional circuits
12:34 The electric force F = q E
12:24 Voltage along finite paths
19:54 Which electric field E?
25:02 Voltage is more than potential difference
27:03 Voltage = PD is a special case
29:02 Conclusion (not really)
My apologies to all native speakers of the English language for the careless butchering of your mother tongue. There are parts of the speech I cannot understand myself. For example, at 07:24 "ballad" stands for "better"; at 10:32 "modesht" stands for "models"; while at 12:39 "thurshaurt" should mean "it turns out".
But let's be frank: who would get past minute five?
(Anyway, I have corrected the subtitles, so...)
________________________________________
References (the ones that have the same explicit definition of voltage):
Simon Ramo, John R. Whinnery, Theodore VanDuzer
Fields and Waves in Communication Electronics 1e
1965, Wiley
Branko Popovic
Introductory Electromagnetic Engineering
1971, Addison Wesley
Branko Popovic, Zoya Popovic
Introductory Electromagnetics
1999, Addison Wesley
J. A. Brandao Faria
Electromagnetic Foundations of Electrical Engineering
2008, Wiley
Переглядів: 3 478

Відео

Dispelling Lewin's 'KVL paradox': the total electric field
Переглядів 7332 місяці тому
The Romer-Lewin ring is a deceptively simple circuit that has two voltmeters reading two completely different voltages 'across' the same two points. The really interesting thing about it is that both voltmeters are correct! 00:00 Intro 00:39 Non-conservative E field and multivalued V 01:31 Generating the time-vatying flux 03:00 Experimental setting 05:30 Computing the E field 07:49 Ohm's law is...
What happens when you put capacitors around a magnetic core?
Переглядів 4,8 тис.3 місяці тому
It appears that the variable magnetic flux channeled by the toroidal core interacts with the capacitors in the ring linking it to produce a temporal shift for frequencies nearing the threshold of 90 kHz. More precisely, the transition is observed when the flux-capacitors system reaches the frequency of 88 kHz. If you want to see what happens in this circuit the other 364 days of the year, look ...
The Romer-Lewin ring with inductors (part 5b: Seeing the burden voltage on the scope)
Переглядів 2303 місяці тому
Finally! My last video on the inductive ring. Just to show how inserting a resistor in the ring alters the voltages. And to show the burden voltage of the UT61E on the scope. Apart from some added noise (and a different selection of acquisition mode) that has increased the measurement by a couple of millivolts - affecting the smallest voltages the most - all measurements show an agreement with ...
The Romer-Lewin ring with inductors (part 5a: Measuring the current - effects of burden voltage)
Переглядів 2813 місяці тому
Of the different ways to measure the current in this low impedance loop (with my low-cost hobbyist equipment), only the use of a low-value sensing resistor results in a decent accuracy. Shown here are the computations and simulations of the ring with real inductors, when there is a 1 ohm sensing resistor and when an ammeter with an internal resistance of 10.5 ohm is inserted in the loop. The UT...
The Romer-Lewin ring with inductors (part 4c) - Agreement between theory and experiment
Переглядів 3314 місяці тому
Here I show how the computed and simulated values for the voltages of the two branches of the ring are in excellent agreement with the actual values measured by the oscilloscope (and also by the voltmeters, as shown in the other videos). This confirms the hypothesis that the self inductance of the ring itself (at the frequency used in this experiment) has nothing to do with the voltages that de...
The Romer-Lewin ring with inductors (part 5 preview) - Current measure and burden voltage
Переглядів 1824 місяці тому
I tried to measure the current in the ring but it was not as easy as I first thought. The internal resistance of the multimeter in ammeter mode can be too high in a circuit like this, with an EMF of 100 mV and a total impedance of about 10 ohms. When in the mA range, the UT 61E shows a resistance as high as 10 ohms that changes the circuit and alters the measurement. Instead of 9-10 mA (rms), w...
The Romer-Lewin ring with inductors (part 4b) - Branch voltages and a spoiler
Переглядів 1714 місяці тому
...and here I show how the rms voltages measured by the 'fast' voltmeter are the same as those shown by the scope. I also insert a spoiler of the explanation of the apparently paradoxical result of seeing two completely different voltages 'across' the same two points: the voltages in the two branches of the circuit are indeed different (as we will see in a future video) but the voltmeters are n...
The Romer-Lewin ring with inductors (part 4a) - Scope measurements step by step
Переглядів 1884 місяці тому
This is ridiculous. I cannot even join two two minutes videos! I have to split part 4 in three or four parts. This video shows *all* voltages on the scope: the exciting 20 Vpp voltage, the EMF of a single turn, and the voltages along the opposite branches of the ring. While looking at the scope screen with all three signals present with the same phase, some of you might be tempted to draw the c...
The Romer-Lewin ring with inductors (part 3 correction)
Переглядів 704 місяці тому
This should have been part of the fourth video, where I show the agreement of theory and practice, but apparently my PC cannot produce videos bigger than a certain amount of MB, or seconds, or whatever. I need to split each video in many small chunks, and I will eventually give up making videos, but for the time being, here are the voltmeters showing the correct voltages (one of which needs to ...
The Romer-Lewin ring with inductors (part 3 REBOOT)
Переглядів 2284 місяці тому
Turns out it was easier to redo the video using the intended value of 4kHz and 20 Vpp for the exciting voltage. I added scope measurements for...good measure. Yes, my sense of humor is stuck in the previous century. As with all the other videos so far, the purpose here is to show that when a variable magnetic field is present inside the circuit path (the ring with the two inductors) voltage is ...
The Romer-Lewin ring with inductors (part 3)
Переглядів 1025 місяців тому
Oh well, about time I completed the inductive ring series. Too bad I have forgotten that the frequency had to be 4 kHz (not 3!) and the amplitude had to be 20 volts, instead of 10. I redid the video with the correct parameters, it's called "part 3 REBOOT". Nevertheless, it is possible to recover the correction factor for the UT139C from the measurements shown (it is left as an exercise for the ...
The Romer-Lewin ring with inductors (part 3 preview)
Переглядів 136Рік тому
The Romer-Lewin ring with inductors (part 3 preview)
The Romer-Lewin ring with inductors (part 2) - Raising the frequency and calibrating the voltmeter
Переглядів 94Рік тому
The Romer-Lewin ring with inductors (part 2) - Raising the frequency and calibrating the voltmeter
The Romer-Lewin ring with inductors (part 1) - Ideal inductors
Переглядів 71Рік тому
The Romer-Lewin ring with inductors (part 1) - Ideal inductors
The Romer-Lewin ring with inductors (part 0) - Intro and ring self-inductance
Переглядів 74Рік тому
The Romer-Lewin ring with inductors (part 0) - Intro and ring self-inductance
The Romer-Lewin ring with capacitors (part 3)
Переглядів 100Рік тому
The Romer-Lewin ring with capacitors (part 3)
The Romer-Lewin ring with capacitors (part 2)
Переглядів 1412 роки тому
The Romer-Lewin ring with capacitors (part 2)
The Romer-Lewin ring with capacitors (part 1)
Переглядів 2222 роки тому
The Romer-Lewin ring with capacitors (part 1)
The Romer-Lewin ring with resistors: two voltmeters read different voltages
Переглядів 5982 роки тому
The Romer-Lewin ring with resistors: two voltmeters read different voltages
The Romer-Lewin ring: an unlumpable circuit
Переглядів 4182 роки тому
The Romer-Lewin ring: an unlumpable circuit

КОМЕНТАРІ

  • @prbmax
    @prbmax 5 днів тому

    Here from Silicon Soup. Oh, the great Kirchhoff is for the birds debate which perplexed me much and then the internet formed well intentioned camps on both sides. "The Mystery of the Lewin Clock" by Treavor Kearney (high level analysis) here on You Tube was posted a couple days ago. Thanks for your contribution as I found it helpful.

    • @copernicofelinis
      @copernicofelinis 4 дні тому

      Thank you for pointing out the new video by Trevor. It seems he, SiliconSoup and I have 'specialized' in covering different angles of the Lewin ring: Trevor is more focused on areas and geometry, SiliconSoup on charge density, while I prefer to look at fields and measurement loops (this last part will be clear when I'll upload the "What a voltmeter measures" video.)

  • @SiliconSoup
    @SiliconSoup 14 днів тому

    I actually reached the end of the video and found it to be interesting and good. I always enjoy your explanation, and it inspire me to read more. Thank you.

    • @copernicofelinis
      @copernicofelinis 13 днів тому

      Thank you for your kind words, 'armchair fellow' . (For everybody else: 'armchair fellow' is kind of an insider joke. SiliconSoup and a few others will understand it). 😊

  • @muzammalbaig
    @muzammalbaig 17 днів тому

    The voltage is very low in the vocal chords of the speaker😂

  • @mattstroker3742
    @mattstroker3742 18 днів тому

    More volume. Can't listen to this

    • @copernicofelinis
      @copernicofelinis 18 днів тому

      Yeah, I get that a lot. 😭 Out of curiosity, do you think the audio level in my previous video "Dispelling the Lewin KVL paradox" to be acceptable? I was almost shouting, there.

    • @no1unorightnow
      @no1unorightnow 14 днів тому

      @@copernicofelinis I have some ideas for you if you're looking to improve your audio! I'm guessing there's some stuff with your microphone, recording and finalizing process, etc. that can be changed to help the volume out a fair bit.

    • @copernicofelinis
      @copernicofelinis 13 днів тому

      ​@@no1unorightnowI'm open to suggestion. I have already tried 'fixing my audio' using win10's diagnostic tool but to no avail. I am now thinking that my mic got damaged when I left the headset in a room where I used ozone.

  • @CG-rr6yx
    @CG-rr6yx 19 днів тому

    Wasting our time .

    • @copernicofelinis
      @copernicofelinis 19 днів тому

      The answer to the question in the title is in the video linked at the end of the description. ua-cam.com/video/qTzqIYrc_Hs/v-deo.html

  • @westinthewest
    @westinthewest 20 днів тому

    This is probably quite interesting but the voice-over is bad. Can you re-record it? Whispering and mumbling is not satisfactory.

    • @copernicofelinis
      @copernicofelinis 20 днів тому

      I have just finished updating a corrected version of the subtitles. The problem with the voiceover is twofold: first, my abysmal pronunciation (compounded by the fact that the recording was initially meant only to estimate the length of the video); second, the low sensitivity of the mic. I have a Logitech headset that behaved spectacularly years ago in Skype - loud and clear - but as of now with Seven and 10 only produces low levels (something like 1-2 notched over a max of 10) and a lot of noise. I had to record this voiceover with my smartphone, keeping the phone in one hand and a pillow between me and the laptop to muffle the sound of the fans. I also did it in the middle of the night, and my voice was naturally on the whisper side, sorry. I have an updated version of this video with a couple more figures (not important) and corrections on the name Helmholtz and the formula overlay frame, but I don't think I will re-upload it anytime soon. I will be more careful with the audio levels (and try to improve my pronounciation) in the next videos. Try using headphones, they can solve the whispering problem. As for the mumbling, I need to work on that 😢. If the headphones won't do the trick, try the subtitles or better yet the transcript. I still have a couple of typos to catch, but they should help. Oh, and thanks for the feedback.

    • @westinthewest
      @westinthewest 20 днів тому

      Thanks for the detailed reply, and sorry if I was abrupt but it was frustrating to find such interesting content which I had to abort! It made sense when you explained that the audio was recorded at night. This was the only problem for me. If you are able to deliver speech at a daytime volume, any old microphone or smartphone will be adequate. The only fancy addition might be some post-production compression, but a smartphone probably has that built-in. Speech doesn't require much bandwidth, dynamic range or s/n ratio. I was unable to follow the content because of the late night whispering :-)

  • @Ancipital_
    @Ancipital_ 20 днів тому

    Is it possible for you to improve the subtitles? This topic really interests me but my hearing is so bad.

    • @copernicofelinis
      @copernicofelinis 20 днів тому

      Is it possible to do this from within UA-cam? Do you know? I have the whole voiceover in the form of text in a web page but I do not have an ad-free website where to upload it to. But a few months ago I had to open a WordPress account to talk to Simplenote's assistance. If I find where I put the credentials, I might upload it there. PS Thank you for pretending the problem is in your bad hearing and not in my pronunciation ;-) .

    • @Ancipital_
      @Ancipital_ 20 днів тому

      ​@@copernicofelinisno no my friend, you're good! I just came home and tried with headphones, it helps a lot, especially with some of the weird mistakes in UA-cam automatic captions. I'm afraid I don't know anything about how to work with UA-cam's subtitles but I'm sure that someone will read this and can provide some information. Again thanks for the video and as my wife will confirm by how I can get startled when she is "suddenly" behind me, giving me a heart attack, my hearing really is that bad 😅

    • @copernicofelinis
      @copernicofelinis 20 днів тому

      Try now: I edited the subtitles. It should be better, if the changes have been applied.

    • @Ancipital_
      @Ancipital_ 20 днів тому

      ​@@copernicofelinisWhoa, that's a lot better indeed. Thank you so much, I appreciate it dearly.

  • @HIGH_TECH_NEWS
    @HIGH_TECH_NEWS 20 днів тому

    This genius

  • @ThinklikeTesla
    @ThinklikeTesla 22 дні тому

    Excellent. I'll keep this in my back pocket for the next time the "Kirchhoff is for the birds" discussion pops off on UA-cam. :) An example of a non-lumpable circuit at ua-cam.com/video/vzdFEaVYQXk/v-deo.html P.S. @ 22:00 sp "Helmholtz"

    • @copernicofelinis
      @copernicofelinis 21 день тому

      I wish I could say it was a one-off typo, but I just realized I have been spelling "Helmholtz" wrong for YEARS. Possibly decades! I just had one of those moments that in the movies are shown with a progressively closer close-up of the actor's face, while the background expands around it. Thank you so very much for pointing it out. PS I commented (positively) to your video years ago. 😊

  • @Tuong11a
    @Tuong11a 24 дні тому

    Like your books

  • @lokeshvirat5915
    @lokeshvirat5915 25 днів тому

    Can you make a video on energy flow in a circuit??

    • @copernicofelinis
      @copernicofelinis 24 дні тому

      That is on the back burner. Right now I am planning to upload a video on the IEC definition of voltage, another one on what a voltmeter actually measures, and one on when probing does matter and why. Then I would like to do a video on the role of surface and interface charge in establishing the E field inside a circuit. And only after that consider the energy transfer question (which has a lot to do with surface charge and also requires making some complicated figures).

    • @lokeshvirat5915
      @lokeshvirat5915 24 дні тому

      Great and yeah we need to understand fundamentals correctly. most people do just read the definitions and move on i really like this video

    • @lokeshvirat5915
      @lokeshvirat5915 24 дні тому

      And also I never really understood the term "flux". The magnetic flux and mmf stuff if you can try to put it in your bucket list .

    • @copernicofelinis
      @copernicofelinis 23 дні тому

      @@lokeshvirat5915 well, there are two important 'dual' quantities that can be defined on a vector field: a line integral that quantifies how much the vectors of the field are going along a given line (when the line is closed this is the circulation), and a surface integral that quantifies how much the vectors of the field go through a given surface (this is flux). The circulation is associated with the projection of the field along the (local) direction of the path, while the flux is associated with the projection of the field along the direction orthogonal to the (local) surface. There are very powerful theorems that can relate the circulation of a given field (how much goes around a given closed curve) to the flux of a related field (how much goes through the surface delimited by that closed curve). And if you look at Maxwells equations in their integral form, you will see that they relate fluxes and circulations of E and B. But this is too long a story to be told in a comment.

    • @ellipse7
      @ellipse7 22 дні тому

      @@copernicofelinis I'm gonna subscribe and turn on notifications , so I don't miss on this ones

  • @lokeshvirat5915
    @lokeshvirat5915 25 днів тому

    Maybe you need to improve presentation and voice over also

    • @copernicofelinis
      @copernicofelinis 25 днів тому

      I agree 😊. This voiceover was a recording I did to find out what the length of the video would have been. I thought I could do that in 15 minutes, and it ended up being 33 minutes. Once I cut out all the pauses I had already invested too much time in it to throw it away. I'll try to improve my pronunciation and timing in the next video.

    • @Antonio-lp8hx
      @Antonio-lp8hx 25 днів тому

      ​@@copernicofelinis Apart from that, I find your explanations really clear and well backuped with figures and equations. Keep it going!

    • @copernicofelinis
      @copernicofelinis 24 дні тому

      ​@@Antonio-lp8hxthanks.

  • @lokeshvirat5915
    @lokeshvirat5915 25 днів тому

    Amazing explanation sir you're pin pointing what a new learner where get confuse with the abstract concepts

  • @DefaultFlame
    @DefaultFlame Місяць тому

    When I saw you set it to 88kHz my face split into a grin. Thank you for doing this.

  • @jerry-yu7yi
    @jerry-yu7yi 2 місяці тому

    kekistan will rise again

  • @fburton8
    @fburton8 2 місяці тому

    Minor language point: one usually says "deceptively simple".

    • @copernicofelinis
      @copernicofelinis 2 місяці тому

      Thank you. I appreciate corrections. Apologies for mauling your language. 😁

    • @fburton8
      @fburton8 2 місяці тому

      @@copernicofelinis No worries! 🙂

  • @ABaumstumpf
    @ABaumstumpf 2 місяці тому

    "Probing has nothing to do with the voltages being different." As several other people had previously shown - Lewin was completely wrong and it is entirely due to his inadequate sloppy measurement setup. It is not even a hard setup to get right. ua-cam.com/video/JpVoT101Azg/v-deo.html

    • @copernicofelinis
      @copernicofelinis 2 місяці тому

      I am well aware of these "probing done right" videos, and what they try to recover is the path integral of the coulombian electric field alone (the middle picture in the diagram with three rings). This is the conservative **part** of the electric field that admits a scalar electric potential. The problem with this approach is threefold: 1. It describes only "half" of the physical system: one should also give the vector potential A to give the complete picture. 2. It forces you to consider a different physics for loops that link a variable flux and loops that do not, making the physics schizophrenic. In particular one has to invent reasons for the failure of Ohm's law inside every coil. 3. Voltmeters measure the difference on voltage, not the difference in electric potential; it is only in the conservative case that these two quantities coincide. I will show this in the next video. So, no, that is not good probing because they put their probes in the middle of the changing magnetic flux region, and because the quantity they recover tells only half of the story.

    • @ABaumstumpf
      @ABaumstumpf 2 місяці тому

      @@copernicofelinis "It describes only "half" of the physical system:" No, they simply are measuring correctly. "It forces you to consider a different physics for loops that link a variable flux and loops that do not" That is not the case either - they simply show how to measure correctly. "Voltmeters measure the difference on voltage, not the difference in electric potential" Aaaaaannnddd you might want to refresh your memory on that one - the definition of voltage for example - cause that IS the difference in electrical potential. You have not addressed anything but instead are making up ridiculous claims. By now it seems more like you subsrcibed the a false notion of physics and are now becoming more and more reluctant to accept that you were wrong, to the point of throwing a tantrum.

    • @copernicofelinis
      @copernicofelinis 2 місяці тому

      @@ABaumstumpf let's see: what is the potential difference between point A and the top terminal of the smallest resistor, according to Mabilde? How many volts? And what is the resistance of the copper conductor for that same path? Show me how Ohm's law work in this case.

    • @ABaumstumpf
      @ABaumstumpf 2 місяці тому

      @@copernicofelinis "Show me how Ohm's law work in this case." You are the one making the false claims. You are claiming that the most well known theories of electrondynamics are wrong - the burden of proof is on you. What Mabilde has down was simply showing that Lewin had an error in his measuring-setup and if corrected all his spurious claims fall apart. And now you are getting emotional cause you made the same mistake.

    • @copernicofelinis
      @copernicofelinis 2 місяці тому

      @@ABaumstumpf what setup? Look at the diagrams of the two half circuits where I show the computation of voltage as E x length. There are NO PROBES. The reason the voltmeters in the video show the correct values as computed from first principles is because the probing there is correct. But the values of those voltages are computed without the need of any probes. Oh, BTW, Ohm's law works perfectly fine with my values of voltage. No need to invent a different physics. Mine is the physics taught at Berkeley and MIT. ( Invoking the authority principle is a good thing in a UA-cam video because it allows viewers to know that what they see is not some 'original research' but is at least based on real science).

  • @woodcoast5026
    @woodcoast5026 2 місяці тому

    Hi Copernico Concerning why the voltmeters give the correct readings for the resistor voltages. The resistor leads are in the induced electric field and the voltage for the resistors is the sum of induced electric field and the Coulomb field. The voltmeter leads are also in the induced electric field and the voltmeters display the sum of the voltages from the induced electric field along the voltmeter leads combined with the potential difference from the Coulomb field created by the resistance of the resistors.

    • @copernicofelinis
      @copernicofelinis 2 місяці тому

      Each voltmeter shows the line integral of the TOTAL electric field along the path that goes through its probes and its internal resistor. This voltage is the same as the voltage along the tested branch of circuit only if there is no changing magnetic field linked. Each voltmeter is connected to both branches of the circuit: one branch will form a loop with the probes that does not link the core, the other will form a loop that does link the core. The voltage shown by the voltmeter will be equal to the voltage along the branch that does not form a loop around the core. All of this will be shown in the next video. What is important to notice here is that the field is a composition of two components and that this resultant is almost zero inside the conductor portions of the ring (in accordance with Ohm's law).

    • @woodcoast5026
      @woodcoast5026 2 місяці тому

      @@copernicofelinis The low resistance only reduces the Coulomb component along the conductor, it does not reduce the Faraday electric field along the conductor.

    • @copernicofelinis
      @copernicofelinis 2 місяці тому

      ​@@woodcoast5026 no. The coulombian component is almost equal to the induced component . They are both around 1.1V/m, their difference is the negligible 0.0002 V/m that is compatible with the high conductivity of copper. The coulombian electric field (almost completely) obliterates the induced electric field in the conductor.

    • @woodcoast5026
      @woodcoast5026 2 місяці тому

      @@copernicofelinis That does not "obliterate" the Faraday field.The Faraday field is still there. The two components are still there forming the two components of the composition .

    • @copernicofelinis
      @copernicofelinis 2 місяці тому

      @@woodcoast5026 yes, that is correct, but the operation of superposition is something that resides in our heads only. What atoms and electrons 'see' and 'feel' is just the resultant field which is nearly zero. And it is nearly zero because the coulombian field has nearly almost entirely canceled the induced field in the copper. The same happens in the electrostatic case: the electrostatic field in a conductor is exactly zero because the external field has been obliterated by the field component generated by the displaced surface charge. Even more interesting: the interior of the conductor *never* gets a chance to 'see' or 'feel' the external field, because the surface charge kills it before it can penetrate the conductor.

  • @Wil_Bloodworth
    @Wil_Bloodworth 3 місяці тому

    LOL

  • @ugurunver2403
    @ugurunver2403 3 місяці тому

    Very interesting concept. As a short film ofcourse. But i like it very much.

  • @richardbeare11
    @richardbeare11 3 місяці тому

    I love this kind of stuff. Fellow engineer types who know how to havr fun 😁

  • @Jeropa
    @Jeropa 3 місяці тому

    What a waste of time is this videoclip! Conclusion: when you put capacitors around a magnetic core, you need to open a box and the camera goes off. Wow.

    • @merkabaenergy9558
      @merkabaenergy9558 3 місяці тому

      It was an April fools joke lol

    • @richardbeare11
      @richardbeare11 3 місяці тому

      It was a joke lmao 😂

    • @king_wing34
      @king_wing34 3 місяці тому

      Just a joke bro :P

    • @EuzebiuszMarek
      @EuzebiuszMarek 2 місяці тому

      The conclusion is that Copernico went back in time so far that he disappeared. He built a time machine.

  • @piad2102
    @piad2102 3 місяці тому

    What's that flare? A green flare, coming from Mars Kind of a green mist behind it It's getting closer, you see it, Bermuda? Come in, Bermuda, Houston, come in, what's going on? Tracking station 43, Canberra, come in, Canberra Tracking station 63, can you hear me, Madrid? Can anybody hear me? Come in, come in ................

  • @freemanrader75
    @freemanrader75 3 місяці тому

    So what's going on here? I have one of those inside my Variac

    • @copernicofelinis
      @copernicofelinis 3 місяці тому

      Toroidal transformers are everywhere, like parsley. Here it is used as a means to generate a sinusoidally changing magnetic flux. If I had had a powerful enough magnet I could have attached that to my grandma's sewing machine and used that instead.

    • @freemanrader75
      @freemanrader75 3 місяці тому

      @@copernicofelinis so could you theoretically use a sinusoidal alternating magnetic field to replace the moving magnets of a generator? Like make a solid state generator that use 2 alternating sinusoidal magnetic fields at 90* to eachother.

    • @copernicofelinis
      @copernicofelinis 3 місяці тому

      ​@@freemanrader75 I believe the point of a generator with moving magnets is that to convert mechanical energy (used to move the magnets) into electrical energy. If we remove the magnets and replace them with an electrically generated variable field, we end up with a transformer.

  • @copernicofelinis
    @copernicofelinis 3 місяці тому

    The burden voltage on the scope is at 06:30. (I hate videos where you have to search what is promised in the title 😊)

  • @freemanrader75
    @freemanrader75 4 місяці тому

    Looks like a toroidal shaped high frequency transformer or something... What's going on here?

    • @copernicofelinis
      @copernicofelinis 4 місяці тому

      Just showing that when a variable magnetic field is present inside the circuit path (the ring with the two inductors) voltage is no longer uniquely defined, and depends on the path. In this case, it depends on which side of the ring the voltmeter is located. Lookup "Kirchhoff is for the Birds Lewin" to see a full fledged lecture on what is going on.

  • @copernicofelinis
    @copernicofelinis Рік тому

    What is this? It's copper wire in an acetic acid solution (vinegar). Some of the copper atoms passed into the solution and when they form a complex with six water molecules, they absorb light in the orange+reddish portion of the visible spectrum. This absorption makes the solution look bluish, but it also steals the components of light spectrum that would have been reflected by the metal. The result is that, when the copper is seen behind a sufficient volume of solution, the characteristic copper color of... copper disappears and the wire looks silverish. (Better seen when looking from above, as shown on my previous two videos). The effect is better appreciated in real life; for some reason the camera does not capture the striking silverish appearance of the wire.

  • @icey69420
    @icey69420 Рік тому

    wow nice

  • @woodcoast5026
    @woodcoast5026 Рік тому

    @ Copernico Felinis There is no magnetic field in the vicinity of the Voltmeter wires but there is an induced electric field from the solenoid along the voltmeter wires. It causes induction of current to take place at the location of the voltmeter wires and that electric field is present along with the coulomb field originating from around the ring with the capacitors. So the Voltmeters readings are in agreement with the voltages across the capacitors.

    • @copernicofelinis
      @copernicofelinis Рік тому

      That is correct. If you allow me some nitpicking, what is important is that there be no appreciable variable magnetic field *linked* by the measuring loop. There is a changing magnetic field 'in the vicinity' because the core is in the vicinity' of the probes, but the loop formed by voltmeter, probes and 'nearest capacitor' does not link any of it. That is where one CAN APPLY KVL. Notice that each voltmeter can be seen as part of two measurement loops: one is that described above, the other is the loop formed with the 'farthest' capacitor. This second loop, though, does link the variable magnetic field in the core and KVL does not apply. One needs to use Faraday's law to recover the correct voltage across the farthest branch of the ring.

  • @georgejo7905
    @georgejo7905 2 роки тому

    A torroidal transformer ? All the mag flux is contained . If you had used a solenoid then the probes would pick it up

    • @copernicofelinis
      @copernicofelinis 2 роки тому

      With an infinitely long solenoid all the flux is contained as well. With a long enough solenoid the flux in any finite region around the core is negligible (because it spreads out over all space). As a matter of fact, Lewin measured the field around his solenoid and confirmed it is negligible where his ring sits. Anyway, how do you explain these results? There is no B field where the ring is, and the self inductance of the loop is negligible at 50 Hz.

    • @georgejo7905
      @georgejo7905 2 роки тому

      @@copernicofelinis Invoking infinity on a tabletop seems a bit "theoretical". After all we are doing experiments. I might add the explanation is due from the theorist / experimenter. Critics are allowed to criticise.. I should not cite self inductance but rather mutual inductance if there were any flux that is . I did not see lewin actuall measure the flux around his solenoid which was not infinite . Even in gigh school 50 yars ago I did this experiment , I did not know EM theory at 15 hut I studied coils about 10 cm and 10 or so turns . One energised weakly with a signal generator the other on a scope . I picked up plenty of signal at more than 10 .

    • @copernicofelinis
      @copernicofelinis 2 роки тому

      @@georgejo7905 if the ring does not go around the core you won't get any appreciable signal at 50 Hz. I can also devise an experiment where there is no primary coil and the change in magnetic field is caused by a magnet falling through the ring with the resistors. in that case there is not even mutual inductance. Did you read the stack exchange pages I linked in the description of the first video?

    • @georgejo7905
      @georgejo7905 2 роки тому

      @@copernicofelinis Sorry not on stack exchange . No other way to link ? just write it? not sure which video

    • @copernicofelinis
      @copernicofelinis 2 роки тому

      @@georgejo7905 stack exchange is not a video network. You do not need to be part of it to read the pages. The pages I mean are linked in the description of my first video. Just click there and you will read the html page, no subscription or shady s**it required (maybe they try to put cookies like 99% of the websites)

  • @woodcoast5026
    @woodcoast5026 2 роки тому

    What is the resistance of the path along the connections to and from the oscilloscope and through your oscilloscope.

    • @copernicofelinis
      @copernicofelinis 2 роки тому

      Input resistance of the scope is 1 meg; probes and copper interconnects in the ring have negligible resistance. Scope measurements confirm what I have measured earlier (not shown in the video) with my multimeter (10meg minimum input impedance). Even the ESR of these cheap caps (which is usually a few ohms) is not playing a part in this circuit. (The ring with inductors is another thing...)

    • @woodcoast5026
      @woodcoast5026 2 роки тому

      @@copernicofelinis I have made this circuit with resistors. Are the grounds for the channels you are using on your scope for this demonstration electrically connected together by the manufacturer at the scope .

    • @copernicofelinis
      @copernicofelinis 2 роки тому

      @@woodcoast5026 yes, of course. It does not affect the measurements, as can be inferred from the fact that I get the same values when I measure the voltage with a voltmeter (which is floating).

  • @yelloweater5506
    @yelloweater5506 2 роки тому

    Unfortunately I don’t have an oscilloscope and also I think I can’t afford it

    • @copernicofelinis
      @copernicofelinis 2 роки тому

      You can use a voltmeter to measure the AC voltages, like I showed in my other video.

  • @woodcoast5026
    @woodcoast5026 2 роки тому

    Hi Copernico Is the field in the vicinity of your transformer mainly electric or is it a mix of magnetic and electric.

    • @copernicofelinis
      @copernicofelinis 2 роки тому

      Ideally all the magnetic field is inside the toroidal core. The physical circuit (resistors and copper wire) can occupy a region of space where there is no magnetic field at all. (In practice if a current is flowing due to the induced electric field, this current will generate a magnetic field around it, but in this case it is so small that we can neglect it - and in doing so we also neglect the self inductance of the ring)

  • @copernicofelinis
    @copernicofelinis 2 роки тому

    00:00 *Introducing the ring* I built it by soldering two naked strands of copper to a 2.2 ohm resistor on one side, and a 15 ohm resistor on the other side. I added a few soldering blobs here and there to prevent the strand from undoing itself. I left one arc longer than the other, then I cut it so that I could open the ring and then tie it around a transformer core. 00:28 *The toroidal transformer* Here I am measuring the emf due to the magnetic flux linked by one turn around the core of a toroidal transformer. It came out as 372 mV (all measurements are RMS, and I am ignoring the error in the voltmeter reading, for simplicity) Note that the shape of the secondary coil - realized with the probes and closed on the internal resistance of the voltmeter - is irrelevant because all of the flux is confined inside the core. The induced electric field, which runs in circle around the core, behaves as a conservative field in all regions of space that do not contain the core. But does not behave as a conservative field when we go around the core. 00:53 *The open ring around the core* Here I placed the ring around the core, but I left it open and placed the voltmeter across the gap. We are basically closing the loop with the internal resistance of the voltmeter. Let's say its 10 megaohm (it's actually higher when the dial in on the millivolt range for this particular multimeter): the full resistance of the loop becomes a little more than 10 megaohm. The current flowing in the ring with the added voltmeter is 272 mV / 10 megaohm = 27.2 nA What are the voltage drops on the resistors, in this setting? 2.2 ohm * 27.2 nA = 59.84 nV 15 ohm * 27.2 nA = 408 nV Since less than half a millivolt drops along the resistors, the entire emf is basically appearing in the form of voltage drop across the internal resistance of the voltmeter. 01:16 *Closing the ring around the core and using a voltmeter to probe an arc of conductor (nearly 2 inches, or about 5 cm.)* When we close the ring around the core, a much bigger current will flow in the ring than when the 10 meg internal voltmeter impedance was part of it. Iring = 372 mV / (2.2 + 15) ohm = 21.6 mA The computed value agrees with what I measured with a clamp on amperometer at 01:46 . The UT136 gives a reading of 20 mA - close enough to the theoretical value, considering the accuracy of the instrument. This current flowing in 5 cm of copper will drop a voltage of the order of a microvolt, and in fact the voltmeter reads nearly 0V. Now, this is a crucial point in understanding how to correctly apply KVL or Faraday in this circuit: the loop formed by the voltmeter, its two probes and the 5cm arc of conductour is sitting in a region of space that does not link any variable magnetic flux from the transfomer (remember, all the flux is contained in the core and this measurement loop does not run around the core). So *we can apply KVL to THIS loop* : we can consider voltage as path independent in this loop and avoid to refer it to a particular path, by specifying the endpoints only. The voltage along the copper arc between the tips is equal to the voltage along the path going through the probes and the internal resistance of the voltmeter and shown by the instrument. Compare this to the case of open ring: now that the gap has been replaced by 5cm of copper, the voltmeter is no longer able to read the open circuit voltage. Path independance *does not apply to the ring itself* , though, because the ring forms a loop that does run around the magnetic core. 02:05 *And now... the full fledged ring with both voltmeters* Here I placed the probes of both multimeters at the same points on the ring (more or less). Now we can identify six loops of four kinds: the ring itself (running around the core), the external loop formed by both voltmeters and their probes (running around the core), the two measurement loops formed by each voltmeter with the 'distant' resistor (still running around the core), and finally the two measurement loops formed by each voltmeter with the 'near' resistor. Only these last two loops do not run around the core, therefore they do not link any (appreciable) variable magnetic flux - and certainly the do not link the variable magnetic flux that is confined in the toroidal core. It is these two measurement loops (voltmeter UT61E and 2.2 ohm resistor on one side, and voltmeter UT139C and 15 ohm resistor on the other) that we can apply KVL to. This means that the voltage across the internal resistance of each voltmeter is equal to the voltage across (and along) the 'nearest' resistor. And, lo and behold, the UT61E reads 48 mV - what we expect by applying Ohm's law to a 2.2 ohm resistor with a 21.6 mA flowing through it. The other voltmeter, the UT139C reads something between 314 and 329 mV - accuracy is significant less in this instrument - still in line we what we expect when 21.6 mA flow through a 15 ohms resistor. KVL works fine in the outer loops, the ones that are not linking the magnetic flux, but *it is no longer valid in the ring itself* . The ring runs around a variable magnetic flux and therefore the circulation of the (total) electric field is no longer zero. This means that voltage (which is defined as minus the path integral of the total electric field) in the ring depends on the particular path between two points. And sure enough, voltage along the path that goes through the 2.2 ohm resistor (and also along many other paths in the nearby region) is 48 mV, while voltage along a path joining the very same endpoints but going through the 15 ohms resistor is 320some mV. (There is also an inversion of phase, as can be seen if we use an oscilloscope instead of voltmeters like I did in my other potatorama video.) As Lewins says: KVL is for the birds. And it is for the birds in the ring itself, and in all measurement loops that happen to run around the core. We can still apply Faraday, tho.

    • @woodcoast5026
      @woodcoast5026 2 роки тому

      Hello C F, you stated incorrectly that Kirchhoff loop rule can be applied to the meter and adjacent resistor. It cannot , because two conductors in parallel do not form a loop. The rule for them is not Kirchhoff it is the Ohms law for conductors in parallel .

    • @copernicofelinis
      @copernicofelinis 2 роки тому

      @@woodcoast5026 two resistors in parallel form a loop. The rule for parallel resistors comes from applying KVL to such a loop. The voltage across either resistor is the same exactly because of KVL.

    • @woodcoast5026
      @woodcoast5026 2 роки тому

      @@copernicofelinis You just stated "two resistors in parallel form a loop" that is an absurd statement. When two resistors are in parallel the current runs from one end of the split path to the other end of the split path. KVL applies to conductors in SERIES. The meter and resistor are not in series they are in PARALLEL.

    • @copernicofelinis
      @copernicofelinis 2 роки тому

      @@woodcoast5026 a loop is any closed path in a circuit. See here: www.electrical4u.com/nodes-branches-and-loops-of-a-circuit/

    • @woodcoast5026
      @woodcoast5026 2 роки тому

      @@copernicofelinis It is not a loop, it is not a closed path. It goes through the meter and resistor in parallel , and then continues around the ring.

  • @woodcoast5026
    @woodcoast5026 2 роки тому

    Some people mistakenly think there is a circulating current going around and around a loop of the internal resistance of the voltmeter and the resistance of the adjacent resistor. You could DISPROVE this by a demonstration, replacing R1 with two 4.4 Ohm resistors side by side and replacing R2 with two 30 Ohm resistors side by side. Now with that arrangement with 6 resistances the circuit behaves as before and on one side of the circuit there are three resistances, and the concept of the current circulating around the two resistors and then going on through the internal resistance of the voltmeter is impossible.

  • @woodcoast5026
    @woodcoast5026 2 роки тому

    For the Romer Lewin Circuit. Here is a concise explanation for why the voltmeter readings are in accordance with the resistor values. Voltmeters display the value of the electric field that lies along the path of their internal resistance. For the Romer Lewin circuit, Faraday's law dictates that the current always flows in one direction around the flux and so therefore there is a meter/resistor pair in series with another meter/resistor pair, and as the resistor of a pair is in parallel with the meter in that pair the resistor defines the value of the electric field lying along the path of the resistance of the voltmeter in that pair.

    • @biskwit2416
      @biskwit2416 2 роки тому

      I've been going through this without any induced EMF and it doesn't seem to work with just the pure interpretation of Faraday no matter what I try. Because looking at the resistor voltage measurements, which are supposed to be the only voltages in the circuit and hence are truly representative of the actual electric field in the loop. By working out the net electric field in the loop by superposition, adding the electric field contributions of the voltages across the two resistors around the loop. This means the loop electric field actually runs in the OPPOSITE direction to the current and the electric field in the resistors is in the SAME direction as the current. Have I missed something and been a dumb dumb?

    • @woodcoast5026
      @woodcoast5026 2 роки тому

      @@biskwit2416 Current going clockwise. R Right, the current goes into the top of R Right and out of the bottom of R Right. E field high at top of R Right. R Left , the current goes into the bottom of R Left and out of the top of R Left. E field High at bottom of R Left.

    • @biskwit2416
      @biskwit2416 2 роки тому

      @@woodcoast5026 R Right current in top Voltage high, bottom Right low. E-field clockwise. From R right top, Voltage high, along loop, field ant-clockwise.

    • @woodcoast5026
      @woodcoast5026 2 роки тому

      @@biskwit2416 Yes, current flows along an E field from High to Low. There are two fields , one for R Left, and one for R Right.

    • @biskwit2416
      @biskwit2416 2 роки тому

      @@woodcoast5026 Take off R left and short. Just have the right resistor with current clockwise has say PD of 1V running from top to bottom so clockwise. So E-Field is clockwise. Go from top terminal of R at 1V going left around the copper and around to 0V R terminal. Field from 1V to 0V must be anticlockwise. Clockwise through the resistor ant-clockwise in the copper. Is this possible?

  • @woodcoast5026
    @woodcoast5026 2 роки тому

    Maybe a correction at 45 seconds. The internal resistance of the meter is not low. It is very high.

    • @copernicofelinis
      @copernicofelinis 2 роки тому

      That's my broken English. I tried to say: "...and the internal resistance of the meter is the load". It came out as unintelligible maybe, but iirc the automatic translation got it right, despite my pronunciation. (Not so much "portion", which apparently I pronounced "Persian")

    • @woodcoast5026
      @woodcoast5026 2 роки тому

      @@copernicofelinis yes I hear it . Load

  • @SiliconSoup
    @SiliconSoup 2 роки тому

    At 1:40, the voltage across the wire segment is 0V, same as my measurement.

    • @copernicofelinis
      @copernicofelinis 2 роки тому

      Yes, in fact the multimeter shows practically zero. The 0.0004 volt is nearly zero considering the error of measurement and the fact that a couple of inches of copper wire with 20 mA in it will drop a handful of microvolts. I will add some numbers in the description. I wanted to add titles and drawings, but I was not able to install OpenShot (cx_freeze error) so I just stitched the videos together.

  • @woodcoast5026
    @woodcoast5026 2 роки тому

    For the Lewin Circuit. Here is a concise explanation for why the voltmeter readings are in accordance with the resistor values. Voltmeters display the value of the electric field that lies along the path of their internal resistance. For the Lewin circuit, Faraday's law dictates that the current always flows in one direction around the flux and so dictates which resistor defines the value of the electric field lying along the path of the resistance of the voltmeter.

  • @biskwit2416
    @biskwit2416 2 роки тому

    Hi Copernico. I was wanting a little help on the E-field inside a conducting sphere as to why it's zero. I have Feynman's Volume 2 5-8 page 5-5. It's concerning the Gaussian surface used by Feynman as another way to prove it's zero. Do you have the time or the patience?

    • @woodcoast5026
      @woodcoast5026 2 роки тому

      M.I.T. lecture goes into charged sphere in great depth ua-cam.com/video/JhV-GOS4y8g/v-deo.html

    • @biskwit2416
      @biskwit2416 2 роки тому

      Great cheers 👍

  • @biskwit2416
    @biskwit2416 2 роки тому

    Hi Copernico. Thank you for your reply earlier. If this is true then all electronic manuals on transformers need to be re-written. It is still a probing problem imo. If you still have your setup, try the following. With it all setup as previous, unsolder one end of any of the resistors. You are now seeing the induced EMF in the probes, the same voltage that is induced in the loop. If you want to you can remove the ring and just have the probes connected to each other, gnd to gnd tip to tip in the same config as you have. You will get the same result. If you want me to go further in my proof this is just a transformer with an induced EMF then we can go to an 2 turn configuration with taps half way along. This way we can begin to negate the induced EMF in the probes by measuring across adjacent turns in close proximity and you can measure VR1 and VR2 and they will be different to this mid-point measurement, which will be half the induced voltage. I think my maths is right, I haven't tried it yet! Remember that the ground wires are commoned (Is this a word)? at the scope. Made this mistake plenty of times in my haste!!

    • @woodcoast5026
      @woodcoast5026 2 роки тому

      It is not a probing problem. You do not even need any probes. Remove the voltmeters entirely. Ohms law V=I*R. The current through each resistor is the same and the voltages are different, and so no doubt the voltages are different. Use an infrared non contact thermometer to display the temperature of each resistor due to power dissipation. The power formula W=V*I rearranged V=W/I , for the different temperatures present, that confirms the presence of the different voltages.

    • @copernicofelinis
      @copernicofelinis 2 роки тому

      If I had a dime for every time I've read this objection, I'd be running my own Space Agency. :-) Yes, when we break the ring and attach a voltmeter in the gap we just created, we measure (practically almost exactly) the whole EMF. There is no inconsistency at all with the fact that the voltage along the conducting part of the ring - and even the resistors, now - is (almost exactly, because there is a tiny current flowing through the high impedance voltmeter) zero. What the voltmeter on the side of the gap is measuring correctly is the voltage *across* the gap, along any path that does not run around the variable flux region. The crucial point is that in order to be sure the voltmeter is not affected by the changing flux, you must be sure your measuring loop (only that!) does not run around the changing flux region. If you open the ring and you want to correctly measure the voltage *along* the ring with the resistors (or along any path jumping from one end of the cut to the other but running around the magnetic flux region) you must create a measuring loop that comprises this particular path but that does not enclose the magnetic core. In fact, if your measuring loop sits in a region of space free of variable magnetic fields, all the voltages between the same pair of points will be all equal, irregardless of the path of integration. In this setting, the voltage along the probes and the internal resistance of your voltmeter will also be the voltage across the voltmeter and probes, which will also be the voltage along the ring with the resistors. Therefore, by putting the voltmeter on the other side of the gap and the core you can measure the voltage along the opened ring. You get a different voltage from the one across the gap but that's alright: voltage can depend on path. If we remove the resistor an leave a single open circuited coil, the conclusion is that the voltage across the coil is the emf, while the voltage along the coil is (ideally) zero. Again, voltage depends on path, and this dependence is essential in deriving the well known L di/dt formula for the voltage across the terminals of a lumped self-inductance. If you make a multi-turn transformer you will be able to measure different voltages by tapping different turns - but your voltmeter outside the transformer will correctly measure only the voltage *across* the jump between turns, and not the voltage *along* the portion of coil between its tips. If you want to compute the voltage along the coil you need to apply Faraday's law and add (or subtract) to that reading the emf multiplied by the number of times you go around the core. You will get zero in the ideal case, or just a tiny ohmic drop in copper in a real setup. This is why you can't measure any voltage build up in the turns - basically, there is none!

    • @biskwit2416
      @biskwit2416 2 роки тому

      @@copernicofelinisGood day Copernico!! Thank you again and I'm sorry for your frustration in dealing with the same old questions! In saying that, I'm not sure you did what I asked.!!!! The whole crux of the argument seems to be that the scope probes, thus positioned in your experiment, are part of the loop where the field is free of changing flux, hence do not interfere with the results. I asked you to remove the ring totally and just leave the probes in situ, with gnd to gnd where they were originally connected to the ring and the same the other end where you will have to join tip to tip. You will get an EMF similar to what you measured in the ring. That's what I don't get. Does this "pickup" miraculously go away when the probe is attached to the reconnected ring? As far as there being a voltage difference between windings, but no voltage along it, that is something I find unfathomable. I think I'll leave it at that and thank you for your civility and patience. I'm a bit knackered so will read through it again soon! Take care👍

    • @copernicofelinis
      @copernicofelinis 2 роки тому

      @@biskwit2416 and if I had another dime for this, as well, I'd be headed for Mars, not just a mere 100 km above. Don't worry, questions are fine and this is one question one needs to go through to understand the ring. Of course you will read a voltage on the two voltmeters: the emf will be split in the same ratio as the internal resistances of your voltmeters. So, for example with an emf of 1V, if you have a nice voltmeter with 90 meg input impedance and a thrift store voltmeter with just 10 meg input resistance, you will read +0.9V on the first voltmeter and -0.1V on the second one. Again, voltage depends on path. If the voltmeters were identical you would read +0.5V and -0.5V. But here is the catch: with only two voltmeters and no other component, each voltmeter only sees one measurement loop, and that loop goes around the variable flux region. You can't apply KVL to that, and in fact voltages in that loop depend on the path. Incidentally this single loop shows that the multivalued voltage is a properties of the loop alone, and has nothing to do with probing (the probes here are the circuit itself!) When you consider the ring with the resistors and you attach the voltmeters, each voltmeter sees three different measurement loops: one with the nearest resistor (which is magnetic free and for which you can apply KVL); a second one with the distant resistor that goes around the core, and finally a third one with the other voltmeter (this one as well goes around the core). The only correct 'direct' reading is that associated with the first measurement loop. For the other two you need to apply Faraday's law and subtract (or add, depending on the signs) the emf to find the correct voltage along the other branches. The beauty of it is that correct voltage of the near resistor is equal to the 'correct' voltage of the second resistor minus the emf. There are no inconsistencies. The other loops that go around the core are, so to speak, shunted by the nearest resistor and what you measure is the result of the interplay of all branches which is basically identical to the effects of the emf on the ring alone. You can solve the circuit for the different possibilities: circuit without voltmeter, circuit with finite input impedance voltmeters, and even measurements loops alone (without all other components) to see that the complete solution is the result of the interplay of all branches but the only branches that count are basically those with the small resistors. (Do not expect dramatic changes in the currents due to the aforementioned shunting, the currents are all in the nanoamps range) The presence of the voltmeter branches adds a minor loading effect of a few nanovolts (and a change in current of a few nanoamps). Yes, I did solve the circuit in all flavours.

    • @copernicofelinis
      @copernicofelinis 2 роки тому

      @@biskwit2416 heres a little exercise that has no variable magnetic field in it. Consider a 1000V generator with an internal resistance of 50 ohm. Attach a 10 meg voltmeter to its terminals. What do you read? And what is the current through the voltmeter? Now shunt the voltmeter with a 0.5 ohm resistance. Do you still read 1000V? What is the current through the voltmeter? And through the 0.5 ohm resistor? Finally consider only the generator with its resistance and the 0.5 ohm load. Has the voltage and the current in the 0.5 ohm resistor changed much without the voltmeter?

  • @ferferfferf6644
    @ferferfferf6644 2 роки тому

    Hello, I come from electroboom and Dr Lewin videos. I saw you had a very informed opinion about the topic and I would like to read in depth what you have to say. Sadly, it's very uncomfortable (basically impossible) to read all the comments you replied to in their videos so I would appreciate it if you could synthesize it here for me and those who might come to your channel under similar circumstances.

    • @copernicofelinis
      @copernicofelinis 2 роки тому

      Yeah, I could write a book by putting all those comments together. :-] What I know comes from electromagnetism books. Three books in particular have this problem clearly explained. Here are the relevant quotes (from one of my comments in another channel's video, slightly edited to remove non-relevant parts): The first book is *Purcell and Morin* , "Berkeley Physics vol 2: Electricity and Magnetism" Section 7.5 "A consequence of Faraday's law of induction is that Kirchhoff's loop rule (...) is no longer valid in situations where there is a changing magnetic field. Faraday's law has taken us beyond the comfortable realm of conservative electric fields. The voltage difference between two points now depends on the path between them." And then it adds: "problem 7.4 provides an instructive example of this fact." And what is problem 7.4? Lewin's ring, around a toroidal core like the one depicted in this video. The problem is solved in the end of the book. And concurs with Lewin, of course. Let's move to MIT with *Haus and Melcher* , "Electromagnetic Field and Energy". This book is entirely online at web DOT mit DOT edu SLASH 6.013_book SLASH www SLASH (I hate you, youtube!) The chapter on magnetoquasistatics opens with Lewin's ring around a toroidal core. There is even a video demonstration (Demo 10.0.1: Nonuniqueness of Voltage in an MQS System). Again, in line with what Lewin says. The third book has a more applicative slant: *Ramo, Whinnery and vanDuzer* , "Fields and Waves in Communication Electronics". This book explain clearly why voltage has to be multivalued and when and how we can treat it as single valued to simplify our treatment of circuits. It lliterally takes you by hand expanding the integrals and showing you precisely where the L di/dt formula come from. In particular at page 174 of the third edition it says: "let us take a closed integral of electric field along the conductor of the coil, returning by the path across the terminals. Since the contribution along the part of the path which follows the conductor is zero, all the voltage appears ACROSS the terminals." Moreover, on page 179, it explains how in lumped circuit theory we pretend to treat voltage as if it were a potential difference: "In the above we seem to be treating voltage as potential difference when we take voltage of a node with respect to the chosen reference, but note that this is only after the circuit [path] is defined and we are only breaking up the integral of E.dl into its contributions over the various branches. As illustrated in the preceding section, we do have to define the path carefully whenever there are inductances or other elements with contribution to voltage from Faraday's law." Finally, to better understand the difference between voltage and potential difference and how the two concepts are linked in the presence of a variable magnetic field, I would suggest *Popovic and Popovic* , "Introductory Electromagnetics", sec. 14.4 "Potential difference and voltage in a time-varying electric and magnetic field". You can find an extended quote searching "Assigning a notion of voltage even when there is a changing magnetic field" on Electrical Engineering Stack Exchange.

  • @woodcoast5026
    @woodcoast5026 2 роки тому

    Hi Copernico Felinis, Faraday's law mandates that a current that is the product of induction can only flow around a closed loop that is spacially defined such that it encircles the flux that generated it, and in one direction only. In accordance with that law the voltmeters and their wires are parts of different current loops, depending on how they are physically arranged and so different currents flow through the meters. Do you concur.

    • @copernicofelinis
      @copernicofelinis 2 роки тому

      I would say that each voltmeter and its probes are part of multiple loops. The current in the voltmeter is the result of the composition of the effects (not the currents) of all loops. For example, in Lewin's ring, voltmeter A and its probes form three loops: one with R1, one with R2, and one with the other voltmeter. If you consider these loops alone, *deleting the other components* , you get three different currents (namely 0, emf/(R2+Rvm1), emf/(Rvm1+Rvm2) ) which are different from the actual current flowing in the voltmeter when all elements are present (it's a rather complicated function, but we can approximate it with emf/Rvm1 R1/(R1+R2) ). Of course we can look at the measurement loops *with the other elements present* . What we can say in that case is that in the loop with R1 - which does not contain variable flux - we can apply KVL, so the voltmeter will measure the voltage along the branch with R1 correctly (voltage is the same for the path going along the branch and for all paths in the space across the terminals, as long as I stay in the magnetic-free region), while the other loops will produce measurements of the other branches that are altered by the linked emf. The nice part is that these measurements will be altered so as to produce the same voltage associated with the first loop. I've read many times the objection "but the other loops go around the changing flux, so you will induce a current in the probes!!!" The point is that the voltmeter (as well as its probes) will see almost none of that current because it is shunted by the nearest resistor.

    • @woodcoast5026
      @woodcoast5026 2 роки тому

      @@copernicofelinis Hi Copernico. I analysed the circuit again. I worked it out and posted my new comment on the subject in the comment section for this video.